Functions that "cube" to one.

แชร์
ฝัง
  • เผยแพร่เมื่อ 26 ต.ค. 2024

ความคิดเห็น • 130

  • @dhrubasarkar5322
    @dhrubasarkar5322 10 หลายเดือนก่อน +146

    Here's a quicker way:
    We already showed f is increasing, so if f(a) > a, then f(f(a)) > f(a) [by applying f on both sides], which in turn implies f(f(a)) > a. Again applying f on both sides would yield f(f(f(a))) > f(a) > a, which is a contradiction.
    For the other case, if f(a) < a, then by similar argument, you can show that f(f(f(a))) < f(f(a)) < f(a) < a, which is again a contradiction.

    • @tomaszadamowski
      @tomaszadamowski 10 หลายเดือนก่อน +27

      Yeah I'm surprised he seemingly missed that and used IVT instead

    • @dhrubasarkar5322
      @dhrubasarkar5322 10 หลายเดือนก่อน +13

      @@tomaszadamowski yeah, this answer seemed more natural to me.

    • @joelklein3501
      @joelklein3501 10 หลายเดือนก่อน +3

      Truee lol I was sos surprised lmao

    • @Jonasz314
      @Jonasz314 10 หลายเดือนก่อน +4

      Right on! I was thinking about exactly the same way to do it, which is both faster and more intuitive.

    • @pepefrogic3034
      @pepefrogic3034 10 หลายเดือนก่อน +2

      He is just lazy

  • @nonamehere9658
    @nonamehere9658 10 หลายเดือนก่อน +71

    12:14 - why is intermediate value theorem needed here? Don't we just have a < f(a) < f(f(a)) < a (contradiction) from 3 inequalities on the board?

    • @mathboy8188
      @mathboy8188 10 หลายเดือนก่อน +3

      True - and your way is more elegant - but his proof works even if that part of his argument can be done another way w/o using the IVT.

    • @DeanCalhoun
      @DeanCalhoun 10 หลายเดือนก่อน

      this is a more natural solution, but using IVT was cool

    • @HagenvonEitzen
      @HagenvonEitzen 10 หลายเดือนก่อน +2

      @@DeanCalhounUsing IVT means using continuity twice (it was already used to conclude monotonicity), so ... meh

  • @matthiasbergner8911
    @matthiasbergner8911 10 หลายเดือนก่อน +84

    Note that if you replace the real numbers R by the complex numbers C, you do get continuous solutions which are not the identity. Namely, for each of the three solutions z of the cubic equation z^3 = 1, one can define f(x) = x*z with the property f(f(f(x))) = x * z^3 = x.

    • @drdca8263
      @drdca8263 10 หลายเดือนก่อน +4

      Are there any others?
      If we are using functions from Riemann sphere to itself, I suppose we have the ones he mentioned with the (x+A)/(x+B) ...
      And I guess, there would be similar such functions for any 2x2 matrix which cubes to the identity matrix?

    • @ribozyme2899
      @ribozyme2899 10 หลายเดือนก่อน +7

      ​@@drdca8263 If we're on the Riemann sphere, yeah. Then the only holomorphic functions that solve our problem are the Möbius transforms of order 1 and 3. Order 1 is just the identity, and order 3 corresponds to 2x2 matrices with characteristic polynomial x^2+x+1 (cause they need to solve x^3-1=0), meaning matrices with trace -1 and determinant 1.

    • @nikoladjuric9904
      @nikoladjuric9904 10 หลายเดือนก่อน

      Also x^z, where z³=1

    • @ribozyme2899
      @ribozyme2899 10 หลายเดือนก่อน +1

      @@nikoladjuric9904 Can that be defined continuously?

    • @yurenchu
      @yurenchu 10 หลายเดือนก่อน +3

      Also f(x) = A+z*(x) , where z³=1 but z≠1 , and A is any constant.

  • @zygoloid
    @zygoloid 10 หลายเดือนก่อน +20

    Before watching:
    1) f³x=x => f is invertible (the inverse is f²)
    2) An invertible continuous function from IR to IR is either strictly increasing or strictly decreasing.
    3) If f is decreasing then f³ is decreasing, but f³=x is increasing so f is increasing.
    4) a f(a)

    • @ArgumentumAdHominem
      @ArgumentumAdHominem 10 หลายเดือนก่อน +2

      Really nice and clean arguments, thank you. It is really shocking to me though that there exists this magical f(x) = x / (x - 1) which works almost everywhere.

  • @samueldevulder
    @samueldevulder 10 หลายเดือนก่อน +8

    Good place to stop was actually at 11:48 since we have a

  • @enire8477
    @enire8477 10 หลายเดือนก่อน +44

    5:30 You accidentally multiplied the numerator by the numerator, instead of the denominator on this board, the next board does have the correct equations however.

    • @rainerzufall42
      @rainerzufall42 10 หลายเดือนก่อน +5

      That's the moment, when he cleans the board and continues with the correct formula!

    • @holliebuchanan6064
      @holliebuchanan6064 10 หลายเดือนก่อน +1

      @@rainerzufall42 This is how students learn to hate Mathematics, alas. For the love of Mike, admit that a mistake was made and correct it.

    • @iyziejane
      @iyziejane 10 หลายเดือนก่อน

      ​@@holliebuchanan6064 It's good for instructors to admit mistakes, I agree. But also students shouldn't hastily judge the subject of mathematics (or judge themselves in relation to the subject) because of some wrong algebra done on a board, which is usually just done to fill up time. If you care about learning the subject you should be able to fix the calculation, that's what really matters.

  • @AboutMoreGames
    @AboutMoreGames 10 หลายเดือนก่อน +8

    One can also go about this a bit differently, with more of a look towards topology.
    We can take the set {x,f(x),f(f(x))}={a,b,c} with a

    • @19divide53
      @19divide53 10 หลายเดือนก่อน

      Could you explain more why the inverse image of an open set around d is not open?

    • @AboutMoreGames
      @AboutMoreGames 10 หลายเดือนก่อน

      @@19divide53
      The following was my thought process, but now that i think about it again, this argument is very similar to 2 above. As added context: this part was mostly proven by "look at it, it probably holds" (and having the example of the map S^1->(0,1] as a typical discontinuous one in my mind), and thus the argument might have to be refined a bit for the general case, but not by too much.
      The inverse image of d has to be one of the points b or c. If we take an open set U around d, the inverse is also open, and thus intersects both the intervals A and B (in general the connected components) around f^-1(d). Since we know that f is bijective (by the concotonation property) and maps connected components to connected components (by continuity), we know that f restricted to a connected component maps bijectively to another connected component. Now either A or B is mapped into J, but neither of them is in the original interval (-inf, a).

  • @huguesbornet1211
    @huguesbornet1211 10 หลายเดือนก่อน +24

    Of course in Complex set, 120deg rotation with e^i*2pi/3 is one that works.

    • @dawnadmin8119
      @dawnadmin8119 10 หลายเดือนก่อน

      As well as 240 with 4π/3 or -2π/3 and 360 with 0 or 2π (the cube roots of unity).

  • @goodplacetostop2973
    @goodplacetostop2973 10 หลายเดือนก่อน +10

    14:06 The identity function remains undefeated

  • @udic01
    @udic01 10 หลายเดือนก่อน +4

    11:38 the contradiction is right there on the board.
    There was no need to continue for 2 more minutes.

  • @davidgillies620
    @davidgillies620 10 หลายเดือนก่อน +4

    Interestingly, in the case of the discontinuous version, setting A = -2 and solving for constant B gives us -(Golden ratio) and 1/Golden ratio.

  • @noahtaul
    @noahtaul 10 หลายเดือนก่อน +2

    The rational functions that satisfy this are (ax+b)/(cx+d) where a^2+ad+d^2+bc=0. For example, if a=1 and d=4, we can take b=-3 and c=7 to get (x-3)/(7x+4). Then we can just send 1/7 to 1/7 and -4/7 to -4/7.
    It turns out that if we only use rational coefficients, then a function like this can only have “order” equal to 1 (for the identity function), 2 (if a+d=0), 3 (the condition above), 4 (if a^2+d^2+2bc=0), or 6 (if a^2-ad+d^2+3bc=0). Then you can just reassign the values where the denominator is 0.

  • @armanfahradyan6207
    @armanfahradyan6207 4 หลายเดือนก่อน +1

    maybe it's useful to know that the set of the functions (ax+b)/(cx+d) with the operation of composition is isomorphic to the set of 2x2 matrices (a,b, c,d) with ordinary matrix multiplication, so what was done in the beginning was just computing matrix multiplication

  • @CM63_France
    @CM63_France 10 หลายเดือนก่อน +6

    Hi,
    At 12:00 you already have the contradiction because we have f(a) < f(f(a)) and f(f(a)) < f(a) .

  • @TedHopp
    @TedHopp 10 หลายเดือนก่อน +1

    That side trip through the IVT was totally unnecessary. When you start it, you already have everything written on the board to show a contradiction. Specifically, you have that a < f(a) implies f(a) < f(f(a)) which further implies f(f(a)) < a. Chain those together and you end up with a < a. Boom!

  • @nektariosorfanoudakis2270
    @nektariosorfanoudakis2270 10 หลายเดือนก่อน

    That's funny, a couple of weeks ago I realised that all examples of continuous one-to-one functions who aren't strictly monotone are defined on disjoint unions of intervals. So I proved that all continuous one-to-one functions on an interval are either strictly increasing or strictly decreasing. And now I saw it being used here. The proof is as follows for anyone interested:
    Let f: I -> R continuous and one-to-one, where I is an interval of R. Define
    T={(s,t) ε I^2 : s R the *continuous* function Φ(s,t) = f(s)-f(t). Since T is convex, thus path-connected, a version of intermediate value theorem holds; if the sign of Φ changes between two pairs (s',t') , (s'',t'') there is a (s,t) "between" them such that Φ=0. This contradicts that f is one-to-one, thus Φ is either positive or negative for all points in T, so f is either strictly increasing or strictly decreasing.

  • @OvsankaPoutram
    @OvsankaPoutram 10 หลายเดือนก่อน +5

    You cannot prove that f is increasing supposing that f is decreasing. It can be neither increasing nor decreasing

    • @code_pi2289
      @code_pi2289 10 หลายเดือนก่อน +3

      yeah! why is no one seeing this

    • @ScouseRobert
      @ScouseRobert 10 หลายเดือนก่อน +10

      I think it was said that because the function is both invertible (with inverse f∘f) and continuous then it has to be bijective (and thus one to one).
      So it either has to either strictly increase or strictly decrease otherwise at least one value would map to the same number and then could not be sent back to both of them with two more applications of f.

    • @ribozyme2899
      @ribozyme2899 10 หลายเดือนก่อน +1

      He literally wrote on the blackboard, and pointed it out: a continuous invertible function is either strictly increasing or strictly decreasing.

    • @nektariosorfanoudakis2270
      @nektariosorfanoudakis2270 10 หลายเดือนก่อน

      Take an interval I and the function Φ: T -> R, where T is the "lower triangle" of I^2, Φ(s,t)=f(s)-f(t) for (s,t) in T. T is convex, thus path connected, and Φ is continuous, and non-zero if f is "1-1", so it preserves its sign. So f is either strictly increasing or strictly decreasing.

    • @ScouseRobert
      @ScouseRobert 6 หลายเดือนก่อน

      @@ThomasBushnellBSG
      Deleted/edited also. 👍🏻

  • @emanuellandeholm5657
    @emanuellandeholm5657 6 หลายเดือนก่อน

    The functional cube root of the identity function, nice. Composition is such an under appreciated operation in math outreach IMHO. You can find some seriously hard problems just by trying to solve functions like f^x = g(y). Like the functional square root of sin(x), or even solutions to fractional application problems. This is largely untreaded ground in mathematics.

  • @benstucky8414
    @benstucky8414 10 หลายเดือนก่อน

    Here's a neat way to see that f^n(x)=x has no increasing, continuous solutions for n>1, besides f(x)=x:
    Suppose such an f exists. Note that f is a bijection from R to R. Let G be the set of all increasing, continuous, bijective functions from R to R. Note that x belongs to G as well, and this set forms a group under function composition, with identity element x.
    Fix an enumeration of the rationals q_1, q_2, q_3, ....
    Let g and h be distinct elements of G. Note that knowing the value of g on each q_i determines g by continuity. Define "

  • @HagenvonEitzen
    @HagenvonEitzen 10 หลายเดือนก่อน +5

    The example for non-continuous solutions is overly complicated - and as stated not even a function! How to define f(-B)? And how to define f((B^2+A)/(B+1))?
    A much simpler infinite family of non-continuous solution is the following: Let a, b, c be three distinct real numbers. Define f(a)=b, f(b)=c, f(c)=a, and f(x)=x otherwise.

    • @19divide53
      @19divide53 10 หลายเดือนก่อน

      Yes, f(x)=1/x is not a function.

    • @19divide53
      @19divide53 10 หลายเดือนก่อน

      And in fact the function is constructed to be undefined at -B because then it is (vacuously) discontinuous there. It's one of the most natural ways to get a discontinuous function

  • @trueriver1950
    @trueriver1950 10 หลายเดือนก่อน

    0:54 your two fold example could equally well apply to all complex numbers too, and most other domains over which subtraction makes sense.

  • @MrMctastics
    @MrMctastics 10 หลายเดือนก่อน +3

    I would like to see videos on the "snake oil method" for combinatorial identities.
    The derivation of sum of inverse squares = pi^2/6 using 1/(1-xy) integrated over x and y from 0 to 1 would also be nice.
    I would in general like to see some proofs from "proofs from the book" by aigner and ziegler. They are some hard to understand, but extremely interesting proofs for sure!

    • @yavuz1779
      @yavuz1779 10 หลายเดือนก่อน

      He did a video about the second suggestion a while ago but I don't remember the name of it. Other suggestions would be great to see

  • @VaradMahashabde
    @VaradMahashabde 10 หลายเดือนก่อน +1

    Easier method would be to derive a = f(f(f(a))) > f(f(a)) > f(a) > a, which is a contradiction

  • @zh84
    @zh84 10 หลายเดือนก่อน +3

    12:34 Don't we have a contradiction here already? Above we have f(a) < f(f(a)), and below that we have f(f(a)) < a < f(a). If we leave the a out of the second inequality, which we can do as it's a strict inequality, we have f(a) < f(f(a)) and f(f(a)) < f(a). They can't both be true.

  • @KenCite
    @KenCite 10 หลายเดือนก่อน

    I think it’s easier that a

  • @geoffreytrang8670
    @geoffreytrang8670 10 หลายเดือนก่อน +2

    One could more generally consider continuous functions f where f^n = f o f o ... o f (n times) = id. The argument for the case n = 3 could be easily generalized to show that there are no nontrivial solutions for any odd n. In the case of even n, any continuous involution (i.e., f o f = id) would be a solution for n = 2 (and of course also for all other even n). Is there a solution for even n > 2 that is not an involution?

    • @AboutMoreGames
      @AboutMoreGames 10 หลายเดือนก่อน

      I think there is no solution for even n>2 that is not an involution. I'd argue, that if we split the reals into intervals (like i did in my comment), for even numbers we can argue the the outermost intervals have to be mapped to each other, thus giving a pair that is mapped to each other. Thus it has to be an involution, since now all of the points have to be mapped to one other and back again, lest they be mapped to the outermost and "stuck in a loop".

  • @graham741
    @graham741 10 หลายเดือนก่อน

    f(x) is increasing so its invertible so if G is the inverse of f(x) (im on a phone so i cant type the -1 power) then x = G(G(G(x))) by taking G of both sides three times. B/c f(x) = x is the only function with the property f(x) = g(x) = x, we can see that G(G(x)) = x. By taking F of both sides we get G(x) = F(x) which is again only satisfied by f(x) = x

  • @duchamp1916
    @duchamp1916 10 หลายเดือนก่อน

    Sarkovskii theorem (1964): if f:R->R is continous with x0 such that f(f(f(x0)))=x0, then f has periodics points of any period

  • @dmtri1974
    @dmtri1974 10 หลายเดือนก่อน

    Nice excercise!! In oder to avoid Mobious transforms and tedious handling, I would recommend the no continous function f that maps each k . a_1a_2a_3 ........ to the number k.a_3a_1a_2 ........ taking in consideration the 3 cycle (1,2,3).

  • @ArminVollmer
    @ArminVollmer 10 หลายเดือนก่อน

    To whom it may concern: the Mathematica solution in 07:50 is A -> -1 - B - B^2. , and yields True for the three folded substitution, which means Nest[(-1 - B - B^2 + #)/(B + #) &, x, 3] == x // Simplify yields True.

  • @megaclpb2623
    @megaclpb2623 10 หลายเดือนก่อน

    any single-valued function whose graph is symmetric through the x=y line also satisfies f(f(x))=x, since we know (x,f(x)) is a point in the graph we know (f(x),x) is too because it's symmetric, and (f(x),f(f(x))) is also a point on the graph and since it's a single-valued function there can only be one y for each x so we know f(f(x))=x

  • @henrik3141
    @henrik3141 10 หลายเดือนก่อน

    For all odd number of compositions f(x) = x is the only solution. Funnily for even compositions there can be more (not only in the 2 case as seen at the beginning)

  • @funktorial
    @funktorial 3 หลายเดือนก่อน

    for free we get the corollary that this is true on any linearly ordered set (or maybe it's enough to say a partially ordered set whose order is connected? idk) equipped with the order topology; that it's R in particular doesn't matter and that's cool.

  • @addafarouk839
    @addafarouk839 10 หลายเดือนก่อน +1

    I guess there is no need to use the intermediate value theorem, as the inequality swaps for the values f(a) and a, which contradict the fact that the function is increasing.

  • @567secret
    @567secret 10 หลายเดือนก่อน

    Idempotent homomorphisms in abstract algebra can be really interesting and have some niche use.

  • @AndyGoth111
    @AndyGoth111 2 หลายเดือนก่อน

    Consider the discrete domain {0,1,2}. Your f(x) could simply be f(x)=x+1 (mod 3). Slightly more interesting: {0..5} and f(x)=x+2 (mod 6).

  • @bradhoward
    @bradhoward 10 หลายเดือนก่อน

    Do you really need IVT there? Seems like the inequality a

  • @BlueDog15391
    @BlueDog15391 10 หลายเดือนก่อน +1

    A slightly different approach. Assume there is a point x s.t. x ≠ f(x). Then also f(f(x)) ≠ f(x), so x, f(x) and f(f(x)) are three distinct points. Now take the supremum of all r such that intervals B_r = (x-r, x+r), f(B_r), f(f(B_r)) are disjoint. Then some of these three sets should share a boundary point (otherwise you could increase r). Let's say that X and Y=f(X) are the ones that have intersecting boundaries and let s be a point in the intersection. Since s lies in both closures of X and f(X), we have that f(s) lies in cl f(X) and cl f(f(X)), and f(f(s)) lies in cl f(f(X)) and cl f(f(f(X))) = cl X. Since s, f(s) and f(f(s)) are three distinct points, then X, f(X) and f(f(X)) should be three intervals on a line with each two intersecting on the boundaries, which is impossible on a real line.

    • @AboutMoreGames
      @AboutMoreGames 10 หลายเดือนก่อน

      That's a really nice argument

  • @wilderuhl3450
    @wilderuhl3450 10 หลายเดือนก่อน

    This gives matrix multiplication vibes.
    take the column vectors (1,1) and (A,B) as a 2x2 matrix and square it, this will yield (1+A , 1+B) (A+AB, A+B^2). Multiply this matrix by the column vector (x,1) and divide the top entry by the bottom and you’ll get f(f(x))= ((A+1)x+ A+AB)/((B+1)x+A+B^2)

    • @ChaitanyaTappu
      @ChaitanyaTappu 10 หลายเดือนก่อน +1

      Indeed, there is an homomorphism from the group of nonsingular 2×2 matrices to the fractional linear transformations!

  • @louisreinitz5642
    @louisreinitz5642 10 หลายเดือนก่อน

    f(x)=A/x is my favorite of the the functions such that f ( f ( x )) = x
    A, x 0

  • @mstarsup
    @mstarsup 10 หลายเดือนก่อน +1

    Am I the only one shocked by the "increasing" part of the proof? I didn't find even one comment on how this part is flawed. f not being decreasing on R doesn't make it increasing on R... The proof is a bit more complicated than that.

    • @ironbutterfly3701
      @ironbutterfly3701 5 หลายเดือนก่อน

      Yes, but it is continous and invertible, so must be increasing or decreasing. We eliminated decreasing case.

  • @deadfish3789
    @deadfish3789 10 หลายเดือนก่อน

    You can't use decreasing as the opposite of increasing in your contradiction. Some functions are neither. Your proof is still essentially valid though, if you just remove the first line.

  • @__hannibaal__
    @__hannibaal__ 10 หลายเดือนก่อน

    Hello Michael :
    I found f(f) = 1x ; for real number a generalized, but f(f(f)) = 1x is hard, i try before 13 years go. I still think about.
    For f(f)) = 1x, it s for every symmetric function g(x,y) = g(y,x) ; so f is solution of equations g(f,x) = 1;

  • @dogedev1337
    @dogedev1337 10 หลายเดือนก่อน +3

    A visual way of interpreting this: the condition (f○f○f)(x) = x is equivalent to f○f being the inverse of f. This implies that f is a homeomorphism from R to itself (an automorphism in Top), visually this can be interpreted as a function which internally "deforms" R without overlaps. Since R is a line this would visually correspond to "stretching", "shrinking" parts of R as well as possibly shifting and reflecting it. If f reflects the line, then applying it three times would still reflect it, so f can't reflect it in this case. It seems intuitive that applying f a few times won't return R to the original state. More precisely, if you pick and point, repeatedly applying f will always gravitate it towards some fixed point of f, so if after 3 applications of f it must return to its place of origin, it must have never moved.

  • @opensocietyenjoyer
    @opensocietyenjoyer 10 หลายเดือนก่อน

    3:53 why didn't you put f(x) into f•f, but instead put f•f(x) into f? this was much more convoluted this way

  • @leonidostrovskiy7815
    @leonidostrovskiy7815 10 หลายเดือนก่อน

    I tried a linear algebra approach, not sure if it is valid though. The minimal polynomial gives three eigenvalues: 1 and two complex: -120 and +120 degree rotations, so that the only real eigenvector is the identity function ...

  • @MrRyanroberson1
    @MrRyanroberson1 9 หลายเดือนก่อน

    3:58 little did he realize A=B is f(x) = 1 and if (a+1)x + a(b+1) = (b+1)x^2 + (b^2 + a)x then b+1 = 0; b = -1; (a+1)x = (a+1)x works for all a which means the solution is (x+a)/(x-1) = (a-x)/(1-x) so i wonder if (a-x)/(1-x)^n is always valid

  • @RandyKing314
    @RandyKing314 10 หลายเดือนก่อน

    interesting…this sounds like relating the function operation of composition to a hyperoperation of repeated compositions. could we then ask about “composition roots”?

    • @yurenchu
      @yurenchu 10 หลายเดือนก่อน

      Aren't those usually called _eigen functions_ ?

  • @assassin01620
    @assassin01620 10 หลายเดือนก่อน

    11:31
    Couldn't you conclude here that:
    a < f(a) < f(f(a)) < a
    Which means:
    a < a
    Which is a contradiction?

  • @BillRicker
    @BillRicker 10 หลายเดือนก่อน

    Re increasing claim for f( ), didn't explicitly suppose or prove f invertable. Which yes is required for fact (lemma), and excludes most continuous functions seen in Calculus

  • @alnitaka
    @alnitaka 10 หลายเดือนก่อน

    If f(x) = 1/(1-x), then f(f(f(x))) = x. f is continuous everywhere except for x = 1, which causes division by zero. So this proposition requires that f be continuous everywhere.

    • @dr.kraemer
      @dr.kraemer 10 หลายเดือนก่อน

      If you're willing to imagine a special number '∞' with 1/0 = ∞ and 1/∞ = 0 then it even works for x=1. (I know this isn't a proper use of that symbol, but it works out here.)

  • @sushnatasarkar352
    @sushnatasarkar352 10 หลายเดือนก่อน

    N fold composition please

  • @droro8197
    @droro8197 10 หลายเดือนก่อน

    Nice one.

  • @mikeholt2112
    @mikeholt2112 6 หลายเดือนก่อน

    Not sure what the HW is supposed to be since you already proved f is increasing?

  • @Bodyknock
    @Bodyknock 10 หลายเดือนก่อน

    11:15 I think there’s a small mistake. In the video you proved by contradiction that f is increasing, but you didn’t prove it’s strictly increasing. That is, when you assumed f was strictly decreasing (i.e. a f(b) ) you got a contradiction, which means f is increasing but not necessarily strictly increasing (i.e. a>b implies f(a) ≥ f(b), meaning f could be constant in some parts)
    But you then assumed in the next portion that f is strictly increasing in the proof. It doesn’t kill the proof but it does mean a bunch of > signs should be ≥

    • @normanstevens4924
      @normanstevens4924 10 หลายเดือนก่อน

      A function from R to R which has an inverse must be strictly increasing or decreasing. If not strictly in/decreasing then f(a) = f(b) => f^-1(f(a)) = f*-1(f(b)) => a = b.

    • @Bodyknock
      @Bodyknock 10 หลายเดือนก่อน +3

      @@normanstevens4924 The problem didn’t assume the function is invertible though. It only assumed continuity and that composing the function three times returns x.

    • @normanstevens4924
      @normanstevens4924 10 หลายเดือนก่อน +1

      @@Bodyknock f(f(x)) is the inverse of f(x)

    • @Bodyknock
      @Bodyknock 10 หลายเดือนก่อน +2

      @@normanstevens4924 Sure, but again that’s something you derive, it’s not an assumption. (Again, I’m not saying this is a big problem, a couple of small steps wraps it up.)

  • @BenfanichAbderrahmane
    @BenfanichAbderrahmane 2 หลายเดือนก่อน

    The negation of increasing is not decreasing 🤔

  • @aditaggarwal3486
    @aditaggarwal3486 10 หลายเดือนก่อน

    I thought of this a month ago. It took a discord walkthrough.

  • @hybmnzz2658
    @hybmnzz2658 10 หลายเดือนก่อน

    Nice video. You're really digging these rainbow paint thumbnails aren't you?

  • @coolbepis9301
    @coolbepis9301 10 หลายเดือนก่อน

    Bro didn't spot a < a

  • @padraiggluck2980
    @padraiggluck2980 10 หลายเดือนก่อน

    Rotation by 2π/3.

    • @AndyGoth111
      @AndyGoth111 2 หลายเดือนก่อน

      Yes, if complex

  • @nuranichandra2177
    @nuranichandra2177 10 หลายเดือนก่อน

    I often wonder apart from academic purposes what real value do such problems have?

    • @waharadome
      @waharadome 10 หลายเดือนก่อน

      This one seems very useful to me. To have proven knowledge that there's no strange continuous function that will return to the identity

    • @hybmnzz2658
      @hybmnzz2658 10 หลายเดือนก่อน +1

      Stability and chaotic systems are all about repeated applications of a function.

    • @nuranichandra2177
      @nuranichandra2177 10 หลายเดือนก่อน

      @@hybmnzz2658 that is a very useful piece of information. I will check it out for details.

  • @pepefrogic3034
    @pepefrogic3034 10 หลายเดือนก่อน +4

    Wtf. He could use transitivity of

    • @bookert2373
      @bookert2373 10 หลายเดือนก่อน

      OMG, he didn’t produce the tightest proof possible? Oh, the horror!

    • @pepefrogic3034
      @pepefrogic3034 10 หลายเดือนก่อน +1

      @@bookert2373He si getting messier and messier, very disrespectful to do such a shabby job

    • @bookert2373
      @bookert2373 10 หลายเดือนก่อน +5

      @@pepefrogic3034 I don’t mean to be disrespectful, but perhaps you should be looking for a different channel, one that would properly respect you?

  • @debguha8727
    @debguha8727 10 หลายเดือนก่อน

    If fofof=i can we replicate this and get
    fofofofo.........=i.
    If it is true it implies f=i.

  • @MarcoMate87
    @MarcoMate87 10 หลายเดือนก่อน +2

    The "fact" at 8:02 is not really a fact, because it's false. It's indeed true that if f is continuous and invertible on an interval I, then f is also strictly increasing or strictly decreasing; the problem is that the contrary doesn't hold completely. If f is strictly monotone, f is invertible and f is continuous almost everywhere. In particular, If f is strictly monotone f can only have a countable set of discontinuous points, and every discontinuity must be a jump discontinuity. In this problem, we have a continuous f from the start, but it's never stated that f is invertible by the hypothesis: thus, either we add the invertibility hypothesis, to imply that f is strictly monotone, or we assume that f is strictly monotone by hypothesis, without any other information. Indeed, as noticed by others, the continuity of f is unnecessary to prove the fact that f(x) must be equal to x; the only fact that we really use is that f is strictly increasing.

    • @AboutMoreGames
      @AboutMoreGames 10 หลายเดือนก่อน +2

      The invertibility actually follows just from f○f○f=id, as this implies f○f=f^-1 (and thus bijectivity follows)

    • @MarcoMate87
      @MarcoMate87 10 หลายเดือนก่อน

      @@AboutMoreGames You're right, the functional equation f○f○f=id implies that the inverse function of f exists.

  • @alexiavya722
    @alexiavya722 5 หลายเดือนก่อน

    you talk about homework in the video. Is this supposed to be a companion video to a course you teach?

  • @karlbindl7883
    @karlbindl7883 10 หลายเดือนก่อน +24

    But the discussed function is not defined for all real numbers.

    • @shohamsen8986
      @shohamsen8986 10 หลายเดือนก่อน +27

      Doesn't need to be. The point is that there are multiple discontinuous solutions and he is giving u a family of them. These are locally continuous so define a piecewise solution from all of them. If u really want a continuous and invertible solution, then f(x)=x is the only solution.

    • @comexk
      @comexk 10 หลายเดือนก่อน +8

      If you define a piecewise function from multiple functions satisfying this property, the piecewise function won’t necessarily satisfy the property itself.

    • @shohamsen8986
      @shohamsen8986 10 หลายเดือนก่อน +1

      @@comexk Hmm not sure i agree. The requirement is the function satisfy f(f(f(x)))=x. Lets say i find two solutions g(x) and h(x) that satisfy this. Then g(g(g(x)))=x and h(h(h(X)))=X. Now lets define a new function s(x)=g(x) for xa. Then in general s is not continuous. To check if s(s(s(x)))=x, we need to consider two case, xa. For the former, s(x)=g(x) and we know g satisfies this property. For the latter s(X)=h(X) and this too satisfies this property. Thus s also satisfies this property. Infact, since we don't require g in the region x>a and h in the region x

    • @joshuahanson9905
      @joshuahanson9905 10 หลายเดือนก่อน +2

      ​​@@shohamsen8986x

    • @shohamsen8986
      @shohamsen8986 10 หลายเดือนก่อน +1

      @@joshuahanson9905Im not sure i understand your concern. Why does g(x)

  • @hallzy2379
    @hallzy2379 10 หลายเดือนก่อน +1

    11:53 I'm probably missing something, but doesn't 𝑓(𝑓(a)) < 𝑓(a) together with the fact that 𝑓 is increasing, already imply that 𝑓(a) < a? Since if 𝑓 is strictly increasing doesn't 𝑓(u) < 𝑓(v) ⇒ u < v?